What is the Degree of Field Extensions in Quotient Fields?

  • Thread starter Thread starter cap.r
  • Start date Start date
  • Tags Tags
    Degree Field
Click For Summary
The discussion revolves around understanding the degree of field extensions in quotient fields, specifically examining the relationship between the degrees of polynomials f and g in K(t). It is proposed that if [K(t):K(u)] is finite, it equals the maximum of the degrees of f and g. An example is provided where f=t^2+1 and g=t^3+t+1, demonstrating that [K(t):K(u)]=3, which aligns with the stated theory. A suggestion is made to consider a basis for K(t) over K(u) using elements of degree g, although there is some confusion regarding the nature of elements in K(u). The conversation highlights the complexity of the problem while exploring potential approaches to the solution.
cap.r
Messages
64
Reaction score
0

Homework Statement


so this is a challenge problem that I need help getting started with.
given K a field and K(t) a quotient field over K. let u=f/g for f,g in K(t).
IF [K(t):K(u)] is finite then it is equal to max(deg f, deg g). Why is this true?

Homework Equations


K(t)----K(u)----K

[K(t):K] is infinite obviously since t is transcendental over K.

I can use anything up to Galois theory and although we didn't cover splitting fields yet, I don't think he will mind if i use them as long as it helps

The Attempt at a Solution


as an example I came up with this. f=t^2+1, g=t^3+t+1. then it is easy to see that [K(t):K(u)]=3 which is the max(deg f, deg g).
 
Physics news on Phys.org
Have you tried the direct approach?

Suppose without loss of generality that \deg f < \deg g. There is a fairly simple set of \deg g elements of K(t) which you might try to prove is a basis for K(t) over K(u).
 
I am not sure if i understand the question anymore after reading your hint. what do elements in K(u) look like?
 
Question: A clock's minute hand has length 4 and its hour hand has length 3. What is the distance between the tips at the moment when it is increasing most rapidly?(Putnam Exam Question) Answer: Making assumption that both the hands moves at constant angular velocities, the answer is ## \sqrt{7} .## But don't you think this assumption is somewhat doubtful and wrong?

Similar threads

  • · Replies 1 ·
Replies
1
Views
1K
  • · Replies 1 ·
Replies
1
Views
2K
  • · Replies 3 ·
Replies
3
Views
2K
Replies
2
Views
1K
  • · Replies 1 ·
Replies
1
Views
2K
Replies
6
Views
2K
  • · Replies 2 ·
Replies
2
Views
2K
  • · Replies 3 ·
Replies
3
Views
2K
Replies
10
Views
3K
  • · Replies 3 ·
Replies
3
Views
2K